Sorry forgot to post pictures of the question on last post (here there are)
For question 4 and 5 You have to find what the equation would look like on a graph. brainly wouldn't let me post all the answer options for those questions sorry!

Sorry Forgot To Post Pictures Of The Question On Last Post (here There Are)For Question 4 And 5 You Have
Sorry Forgot To Post Pictures Of The Question On Last Post (here There Are)For Question 4 And 5 You Have
Sorry Forgot To Post Pictures Of The Question On Last Post (here There Are)For Question 4 And 5 You Have
Sorry Forgot To Post Pictures Of The Question On Last Post (here There Are)For Question 4 And 5 You Have
Sorry Forgot To Post Pictures Of The Question On Last Post (here There Are)For Question 4 And 5 You Have

Answers

Answer 1

Answer:

See below for answers and explanations

Step-by-step explanation:

Problem 1

[tex]x=-3+2cos\theta,\:y=5+2sin\theta\\\\x+3=2cos\theta,\: y-5=2sin\theta\\\\(x+3)^2=4cos^2\theta,\: (y-5)^2=4sin^2\theta\\\\(x+3)^2+(y-5)^2=4cos^2\theta+4sin^2\theta\\\\(x+3)^2+(y-5)^2=4(cos^2\theta+sin^2\theta)\\\\(x+3)^2+(y-5)^2=4(1)\\\\(x+3)^2+(y-5)^2=4[/tex]

Thus, the first option is correct. Trying all the other options will not get you the desired rectangular equation.

Problem 2

[tex]x=3-6cos\theta,\: y=-2+3sin\theta\\\\x-3=-6cos\theta,\: y+2=3sin\theta\\\\\frac{x-3}{-6}=cos\theta,\: \frac{y+2}{3}=sin\theta\\ \\ \frac{(x-3)^2}{36}=cos^2\theta,\: \frac{(y+2)^2}{9}=sin^2\theta\\ \\ \frac{(x-3)^2}{36}+\frac{(y+2)^2}{9}=cos^2\theta+sin^2\theta\\\\ \frac{(x-3)^2}{36}+\frac{(y+2)^2}{9}=1[/tex]

Therefore, the first option is correct. This equation is in the form of an ellipse with a horizontal major axis length of 12 (half is 6) and a vertical minor axis length of 6 (half is 3), with its center at (3,-2).

Problem 3

Not sure which equation needs to be used for this problem

Problem 4

[tex]x=-7cos\theta ,\:y=5sin\theta\\\\-\frac{x}{7}=cos\theta,\: \frac{y}{5}=sin\theta\\ \\ \frac{x^2}{49}=cos^2\theta,\: \frac{y^2}{25}=sin^2\theta\\ \\\frac{x^2}{49}+\frac{y^2}{25}=cos^2\theta+sin^2\theta\\ \\ \frac{x^2}{49}+\frac{y^2}{25}=1[/tex]

This equation is in the form of an ellipse with a horizontal major axis length of 14 (half is 7) and a vertical minor axis length of 10 (half is 5). See attached graph.

Problem 5

Eliminate the parameter:

[tex]x=-t^2-2,\:y=-t^3+4t\\\\x+2=-t^2\\\\-x-2=t^2\\\\\pm\sqrt{-x-2}=t\\\\y=-t^3+4t\\\\y=-(\pm\sqrt{-x-2})^3+4(\pm\sqrt{-x-2})[/tex]

Attached below is the graph of the curve, which corresponds with the first option.

Sorry Forgot To Post Pictures Of The Question On Last Post (here There Are)For Question 4 And 5 You Have
Sorry Forgot To Post Pictures Of The Question On Last Post (here There Are)For Question 4 And 5 You Have

Related Questions

Multiply 6−√5 by its conjugate and simplify

Answers

[tex]6 - \sqrt{5} \times \frac{6 + \sqrt{5} }{6 + \sqrt{5} } [/tex]

[tex] \frac{(6 - \sqrt[]{5})(6 + \sqrt{5} ) }{6 + \sqrt{5} } = \frac{36 - 5}{6 + \sqrt{5} } = \frac{31}{6 + \sqrt{5} } [/tex]

Elise had 1/4 of an apple pie. She gave 1/4 of what she had to her friend Danny. What fraction of the whole pie did Danny get?

Answers

Danny had 1/4 of the pie

The answer to your question is =

Danny got 1/16 of the whole pie

what scale factor is used to draw the longer trapezoid?

Answers

What scaled factor is used to draw the longer trapezoid look it up online

Measurements of angles

Answers

Answer:

x = 60

Step-by-step explanation:

Since this triangle is an isosceles triangle, the angle next to the 60 degrees is 60 as well. The interior angles of a triangle adds up to 180 degrees, so:

60 + 60 + x = 180
120 + x = 180
x = 60
Hope this helps :)

I need help with this question really badly please help.

Answers

D: of ;;;9d 38383 valise irl for 1

Each marble bag sold by Lashonda's Marble Company contains 8 orange marbles for every 5 red marbles. If a bag has 45 red marbles, how many orange marbles does the bag contain?

Answers

Answer:

72 orange marbles

Step-by-step explanation:

First, divide 45 by 5 to determine how many red units there are. Then, use that number (9) to multiply with 8 to get the number of orange marbles, which is 72.

Hopefully this helps- let me know if you have any questions!

Find the derivative of the image below

Answers

[tex]~~~\dfrac{d}{dx} f(x) \\ \\=\dfrac{d}{dx} \left(x^3 \cdot \log_6 x \right)\\\\=x^3\dfrac{d}{dx} \left(\log_6 x\right) + \log_6 (x)\dfrac d{dx} x^3 \\\\=x^3 \cdot \dfrac 1{x \ln 6} + \log _6 (x) \cdot 3x^2\\\\=\dfrac{x^2}{\ln 6} + \log_6(x) \cdot 3x^2\\\\\\=x^2 \left(\dfrac 1{\ln 6} + 3\log_6 x \right)[/tex]

[tex]\text{The derivative is now in a form of}~ x^A (B + C \log_D (x))[/tex]

Trigonometric equations
4sin^2(theta) + 4 = 5

Answers

Answer:

Θ = (π/6) + πn

Θ = (5π/6) + πn

Step-by-step explanation:

4sin²Θ + 4 = 5

            -4    -4

4sin²Θ = 1

÷4         ÷4

sin²Θ = (1/4)

√sin²Θ = √(1/4)

sinΘ = (1/2), (-1/2)

-------------------------

Θ = arcsin (1/2)

Θ = (π/6)

to find the quadrant subtract π

Θ = π - (π/6)

Θ = (5π/6)

Find the period

2π / |b|

b = 1

2π/1 = 2π

The sin Θ function is 2π, so values will repeat 2π in both directions.

Θ = (π/6) + 2πn (n is the variable)

Θ = (5π/6) + 2πn

-------------------------------------------------------------------------------------------------------

sin Θ = (-1/2)

Θ = arcsin (-1/2)

Θ = (-π/6)

To find the second function add π

Θ = 2π + (π/6) + π

Θ = (7π/6)

Find the period

2π/|b|

2π/1

(-π/6) + 2π

2π       6         π

----- ×  ----- -  -----

1           6        6

Θ = (11π/6) will repeat every 2π in both directions

-------------------------------------------------------------------------------------------------------

Θ = (π/6) + 2πn

Θ = (5π/6) + 2πn

Θ = (7π/6) + 2πn

Θ = (11π/6) + 2πn

(π/6) + π = (7π/6)

(5π/6) + π = (11π/6)

Θ = (π/6) + πn

Θ = (5π/6) + πn

----------------------------------------------------------------------------------------------------------

I hope this helps!

A container holds 4 gallons of lemonade. How much is this in pints

Answers

Answer:

i think I'd be / the answer is 46

Sanjay is standing on level ground 70 meters away from a flag pole. The angle of elevation to the flagpole is 38 degrees. The height of Sanjay's eyes are 167cm from the ground. How tall is the flag pole in meters?

Answers

The height of the pole is  53 m

What is the height of the pole?

The angle of elevation is the angle formed between a horizontal line and a line of sight or an upwardly inclined line from an observer to an object or point. It is used to describe the angle at which an object is viewed from a particular position

Let the height of the pole be x

We know that;

Tan 38 = 1.67 + x/70

Tan 38 * 70 = 1.67 + x

x = (Tan 38 * 70 ) - 1.67

x = 53 m

Thus we can see that the height of the pole is about 53 m

Learn more about angle of elevation:https://brainly.com/question/21137209

#SPJ1

What is the mean absolute deviation of the following set
of data?
9
2
9
9
5
4.
9
1
4
10

Answers

Answer:

  3

Step-by-step explanation:

You want the mean absolute deviation of the set of numbers ...

  {9, 2, 9, 9, 5, 4, 9, 1, 4, 10}

MAD

The mean of this 10-number set of data is 6.2. The mean absolute deviation (MAD) is found by subtracting that from the members of the dataset, taking the absolute values of the result, and finding the means of those.

The attachment shows those calculations.

The MAD of this set of data is 3.

__

Additional comment

The period (decimal point) in the column of numbers suggests this is a 6-number dataset, and the last 4 numbers are answer choices. If that is the case, none of those answer choices is correct. That is why we assumed that this problem has a 10-number dataset.

When you're calculating this by hand, you only need to consider the values for which the deviation is negative. The MAD is twice their total absolute value, divided by the number of numbers in the dataset. Here, that is 2(4.2+2.2+5.2+2.2+1.2)/10 = 3. (Numbers 2, 5, 4, 1, 4 are less than the mean of 6.2.)

<95141404393>

Which polynomial function could be represented by the graph below?

Answers

Answer:

C

Step-by-step explanation:Only one that makes sence just look at the graph

the sum of 3x and 1 is greater than-5 and less than or equal to 10

Answers

Answer:

x is greater than -2 and less than or equal to 3

Step-by-step explanation:

so the 3x + 1 is greater than -5

so 3x is greater than -6

we divide both sides by 3

then we find out that x is greater than -2

we also know that 3x + 1 is less than or equal to 10

so 3x is less than or equal to 9

we divide both sides by 3

then we find out that x is less or equal to 3

so we can say that x is greater than -2 and less than or equal to 3

What inverse operation will be used to undo the volume of Julie's jewelry box to find its edge lengths?

Volume formula for a cube is: volume = edge ^ 3

Answers

Answer:

Cubed root

Step-by-step explanation:

Do the inverse of cubed, cubed root

$1 = £0.73. Change $346 into £

Answers

Answer:

£252.58

Step-by-step explanation:

$346 × £0.73/$1 = £252.58

Please rate!! I hope this helps!!

Determine the value(s) for which the rational expression 10w−52w2+11w+5 is undefined. If there's more than one value, list them separated by a comma, e.g. w=2,3.

Answers

An expression is defined as a set of numbers, variables, and mathematical operations. The value(s) for which the rational expression (10w−5)/(2w²+11w+5) is undefined is -1/2 and -5.

What is an Expression?

In mathematics, an expression is defined as a set of numbers, variables, and mathematical operations formed according to rules dependent on the context.

The value for which the function will not be defined is the value of w which will make the denominator of the rational function zero. This values can be found by factorizing the denominator of the function. Therefore,

2w² + 11w + 5 = 0

2w² + 10w + 1w + 5 = 0

2w(w+5) + 1(w+5) = 0

(2w+1)(w+5) = 0

w = -1/2 , -5

Hence, the value(s) for which the rational expression (10w−5)/(2w²+11w+5) is undefined is -1/2 and -5.

Learn more about Expression:

https://brainly.com/question/13947055

#SPJ1

A cone has a hight of 18 meters and a radius of 10 meters what is its volume use 3.14

Answers

[tex]\textit{volume of a cone}\\\\ V=\cfrac{\pi r^2 h}{3}~~ \begin{cases} r=radius\\ h=height\\[-0.5em] \hrulefill\\ r=10\\ h=18 \end{cases}\implies V=\cfrac{\pi (10)^2(18)}{3}\implies \stackrel{using~\pi =3.14}{V=1884}[/tex]

Determine whether the statement makes sense or does not make sense, and explain your reasoning.
A candidate received the majority of first-place votes and lost the election.
Choose the correct answer below.

A. The statement does not make sense because the candidate with the majority of the first place votes using the plurality method is the winner.
B. The statement does not make sense because the candidate with the majority of the first place votes in an election is always the winner.
C. The statement makes sense because the Borda count method using preference ballots can result in a situation where this can happen.
D. The statement makes sense because the plurality method can result in a situation where this can happen.

Answers

A candidate received the majority of first-place votes but loses hows that A. The statement does not make sense.

What is plurality voting?

It should be noted that plurality voting simply means when each voter is allowed to vote for only one candidate.

In this case, a candidate received the majority of first-place votes will win the election. Here, the majority of the first place votes using the plurality method is the winner.

Learn more about votes on:

https://brainly.com/question/3710029

what is the length of the arc with a measure of 200 degrees in a circle with a radius of 31 inches?

Answers

Answer:

403π/9 or 140.67

Step-by-step explanation:

First, you have to find the circumference of the circle

Circle circumference equation:

Circumference = dπ (Circumference = diameter x pi)

You have a radius of 31 which is equal to 62 diameter (diameter is double the radius)

So:

C = 62π

Now, you need to know that a full circle is 360 degrees and we want only 200 degrees of that,

So in other words, you want 200/360 of the circle's circumference.

You can find the measure of the 200 degree arc by doing

62π x [tex]\frac{200}{360}[/tex] = [tex]\frac{403pi}{9}[/tex] or 140.67

Please help me i need help!!!

Answers

Answer:

sorry cant help with that

Step-by-step explanation:

When asked to factor the trinomial x^2 - 18x + 81, a student gives the answer (x - 9)(x + 9). Which of the following statements is true?

A. The answer is incorrect; the trinomial cannot be factored.

B. The answer is incorrect; the plus sign should be a minus sign.

C. The answer is incorrect; the minus sign should be a plus sign.

D. The answer is correct.

Answers

Answer:

B.The answer is incorrect; the plus sign should be a minus sign.

Step-by-step explanation:

[tex]x^2-18x+81\\\\=x^2-2\cdot 9x +9^2\\\\=(x-9)^2\\\\=(x-9)(x-9)[/tex]

Amy joined a gym with a $77 enrollment fee. The equation below can be used to find the total Amy has paid, y, if she has belonged to the gym for x months.

y = 44x + 77

What does the 44 represent in the equation?
A.
the amount for x months of membership
B.
the cost to join
C.
the number of months of membership
D.
the monthly dues

answer
the monthly dues

Answers

Answer:

44 represents months

x*y

$77

x

x*y

Which symbol < > = will make the comparison true 4/6 and 4/8

Answers

Answer:

4/6 > 4/8

Step-by-step explanation:

To see if the following is true:

We can cross multiply:

[tex]\frac{4}{6}\neq \frac{4}{8}[/tex]        {4* 8 = 32}      {4*6=24}

Thus,

[tex]\frac{4}{6}=32[/tex]

[tex]\frac{4}{8}=24[/tex]

Hence, [tex]\frac{4}{6} > \frac{4}{8}[/tex]

We can also turn them into decimal:

[tex]\frac{4}6=0.6667[/tex]

[tex]\frac{4}{8}=0.5[/tex]

0.5 is NOT greater than 0.6667 which also means that 4/8 is NOT greater than 4/6.

~lenvy~

what is 1/4 - 41 1/4 + 3

Answers

Answer:

-38

Step-by-step explanation:

i hope this helps you

good luck on your quiz or lesson!

Answer:

-38

Step-by-step explanation:

[tex]\frac{1}{4} = 0.25[/tex]

[tex]\frac{1}{4} -41\frac{1}{4} +3[/tex]

[tex]0.25-41.25+3[/tex]

[tex]-41+3[/tex]

[tex]-38[/tex]

Write an explicit formula for an; the nth
term of the sequence 6, 12, 18, ....

Answers

Answer:

F(n) = 6n

Step-by-step explanation:

An explicit sequence goes by the pattern: A0 + (n-1)d, where A0 is the starting term, n is the nth term in the sequence, and d is the common difference. The explicit formula for this sequence is F(n) = 6 + 6(n-1), or 6n

The answer your looking for is 6n, it starts with six and goes up by six each time.

Diego's recipe calls for three times as much oregano than Tracy's recipe. If Tracy only uses 0.75 tablespoons of oregano in her recipe, how many does Diego use?

Answers

Answer:

2.25 tablespoons of oregano

Step-by-step explanation:

as Diego use 3x more than Tracy, and you know that Tracy use 0.75 tablespoon, so 0.75 x 3 and you will get 2.25

If the difference of 3 / 4 and 1 / 6 of a number is 7, find the number and also verify your answer.

Answers

The computation of the equation illustrates that the number will be 46.4.

How to solve the equation?

Let the number be represented as x.

Therefore, the the difference of 3 / 4 and 1 / 6 of a number is 7 will be computed as follows:

(1/6 × x) - 3/4 = 7

0.167x = 7 + 3/4

0.167x = 7.75

x = 7.75/0.167

x = 46.4

Therefore, the number is 46.4.

Learn more about equations on:

https://brainly.com/question/2972832

Arc cd located on circle a has a central angle of 135 the radius is the circle is 24 centimeters what is the length of arc cd

Answers

[tex]\textit{arc's length}\\\\ s = \cfrac{\theta \pi r}{180}~~ \begin{cases} r=radius\\ \theta =\stackrel{degrees}{angle}\\[-0.5em] \hrulefill\\ r=24\\ \theta =135 \end{cases}\implies s=\cfrac{(135)\pi (24)}{180}\implies s=18\pi ~cm[/tex]

Answer:

B) 18π cm

Step-by-step explanation:

Step 1

Given:

   Central angle θ=135°

    Radius of circle r=24 cm

    Length of arc CD =θ360°(2πr)

Step 2

  l=135°360°×2π×24

    l=38×2×π×24

    l=3×2π×3

      l=18π cm

Therefore

  Length of arc CD =18π cm

Thus, option B is correct.

40 Points! If AD is a diameter and m∠C = 112°, what is the measure of ∠A?

Answers

Both are supplementary

<A+<C=180112+<A=180<A=180-112<A=68

For a sample of n = 16 scores, what is the value of the population standard deviation (o) necessary to produce each of the following standard error values?
a. Ом = 8 points
b. Ом =4 points
с. Ом = 1 point

Answers

Using the Central Limit Theorem, it is found that the values of the population standard deviation [tex]\sigma[/tex] needed are given by:

a) [tex]\sigma = 32[/tex]

b) [tex]\sigma = 16[/tex]

c) [tex]\sigma = 4[/tex]

What does the Central Limit Theorem state?

By the Central Limit Theorem, the sampling distribution of sample means of size n has standard error [tex]s = \frac{\sigma}{\sqrt{n}}[/tex].

In this problem, we have that n = 16.

Item a:

s = 8, hence:

[tex]s = \frac{\sigma}{\sqrt{n}}[/tex]

[tex]8 = \frac{\sigma}{\sqrt{16}}[/tex]

[tex]\sigma = 32[/tex]

Item b:

s = 4, hence:

[tex]s = \frac{\sigma}{\sqrt{n}}[/tex]

[tex]4 = \frac{\sigma}{\sqrt{16}}[/tex]

[tex]\sigma = 16[/tex]

Item c:

s = 1, hence:

[tex]s = \frac{\sigma}{\sqrt{n}}[/tex]

[tex]1 = \frac{\sigma}{\sqrt{16}}[/tex]

[tex]\sigma = 4[/tex]

More can be learned about the Central Limit Theorem at https://brainly.com/question/24663213

Other Questions
weights should be lifted lowered and slowly to _____ injury and maximize effectiveness. how does lasso tool and dodge tool work in photoshop? Which statement best summarizes the economic theories of John Maynard Keynes? Ben runs 3/4 mile each day.How many Miles Does ben run in 12 days Trigonometric equations 4sin^2(theta) + 4 = 5 A students work to simplify a radical is shown below. Select the statement which best applies to the sample mathematical work. where the French, Dutch and English lived? Jennifer has $65 to spend on pants and shirts. She buys a pair of pants for $25 and buys shirts that cost $11 each. What is the greatest number of shirts Jennifer can buy? Enter your answer in the box. What does it mean if someone say Im not rich but I have money Help please!! Thank you!! :)Let m = x^2+3Which equation is equivalent to (x^2+3)^2+7x^2+21=-10 in terms of m?A. m^2+7m+10=0B. m^2-7m+10=0C. m^2+7m+31=0D. m^2-7m+31=0 In a mouse, the axis that determines the body pattern from the nose to the tip of the tail is. Which of the following can cause beach erosion? Select the two correct answers. (1 point)-ocean currents-shifting shorelines-sandbars-dune vegetation-storm waves The equilibrium for the reaction between (ch3)2nh a weak base How does hemingway's decision to end the story of the major returning to the hospital after his wife's death contribute to the story's impact? Hi please help me me with this thank you so much Which 18 events from 1952-2018 most significantly shaped present-day American society? You can buy 7 pairs of jeans for $108.50. how many pairs of jeans can you get for $387.50? with the exception of one or two news stations, there is not much credible evidence that any particular mainstream news source is explicitly biased in one direction or the other in terms of its news coverage. Suppose that 25 inches of wire costs 75 cents.At the same rate, how much (in cents) will 21 inches of wire cost? Help please :3 What is the function rule for the table below?